magnumlifestyle
Thanks Received: 1
Forum Guests
 
Posts: 7
Joined: June 24th, 2010
 
 
 

Q15 - Grow-Again ointment is

by magnumlifestyle Thu Jul 15, 2010 9:38 pm

Hi!

I had a quick question regarding 15, a Strengthen question.

The question basically asks to strengthen the conclusion that offering the rebate on the product will not increase sales and consequently could be unprofitable for the manufacturer.

The correct answer is (C).

(C) The rebate, if offered, would not attract purchasers who otherwise might not use Grow-Again.

My question is this: When offered, the rebate will not be attracting customers who already might not buy the product. So how does this fact strengthen the conclusion?

The purchasers might already not use Grow-Again -- so how does the fact that the rebate won't attract those people who won't use the product anyway strengthen the conclusion?

I'll really appreciate any insights you might offer.

Thanks!
User avatar
 
ManhattanPrepLSAT2
Thanks Received: 311
Atticus Finch
Atticus Finch
 
Posts: 303
Joined: July 14th, 2009
 
 
 

Re: Q15 - Grow-Again ointment is

by ManhattanPrepLSAT2 Fri Jul 16, 2010 2:32 pm

Hello magnum lifestyle! --

Let's think about it this way --

How would the rebate increase sales and profitability?

Mostly likely with some combination of the following --

1) The rebate might get current customers to buy more ointment

2) The rebate might attract new customers who will buy ointment

They've given us information that hints at the fact that current customers will not buy more ointment (doesn't help to go beyond the recommended dose). Therefore, the company can probably only increase sales if they attract new customers.

The author's opinion is that the rebate won't increase sales or productivity. This opinion can be strengthened if it can be shown that the rebate won't attract new customers. (C) addresses this issue.

Hopefully that helps, and please let me know if you have any further questions!
 
skapur777
Thanks Received: 6
Atticus Finch
Atticus Finch
 
Posts: 145
Joined: March 27th, 2011
 
 
 

Re: Q15 - About Grow-Again Ointment

by skapur777 Wed Apr 13, 2011 10:12 pm

Here is my answer break-down since I got this wrong...

A- why couldn't this be correct? People think that by applying more it will be more effective, its not more effective because of the stimulus info and thus they don't buy it anymore...rebate doesn't help that..right?
B- or this one, which i picked. people use it, some apply drops and drops of it and it doesn't work as well for them so they say screw it, not buying again. how will a rebate help that?
C- correct answer, i can see why but im confused about the first two
D-eliminated quickly
E- doesn't really matter, per unit cost could be cheap...

I think the overarching problem here is if i can infer these things for stuff like answer choice A/B. Because honestly im having trouble knowing when i can or cannot bring in outside knowledge..
User avatar
 
ManhattanPrepLSAT2
Thanks Received: 311
Atticus Finch
Atticus Finch
 
Posts: 303
Joined: July 14th, 2009
 
This post thanked 1 time.
 
 

Re: Q15 - About Grow-Again Ointment

by ManhattanPrepLSAT2 Tue Apr 19, 2011 1:55 pm

Based on what you wrote, I have two suggestions for you:

1) Make sure you understand that a "strengthen" answer will almost always fall BETWEEN your supporting premises and your conclusion.

Here the support is: We have a product for which using more isn't any more effective.

The conclusion is: Therefore, selling for cheaper won't make people buy any more product, and will make it so we lose money.

In reaching this conclusion from this evidence, the author is failing to consider many things: perhaps new customers will buy the product, or perhaps people will buy even more of it even though the manufacturer tells them using more won't be any more helpful. There are some significant gaps between this premise and this conclusion, and the right answer needs to help fill in these gaps.

2) Make sure you understand your job in this question, which is to STRENGTHEN the idea that the company will NOT sell more products, and will NOT be preferable.

Let's evaluate (A) -- does this help strengthen the idea that the company WON'T sell more products? No. In fact, I'd say (A) plays the exact OPPOSITE role -- it gives us a reason to believe they WILL sell more with the rebate.

(B) is a much more attractive answer, but again does it give us a reason to believe that the company WON'T sell any more products with the rebate? No. If I said something like "lowering the price of the candy-bar will/won't increase sales" an answer that says "some people like candy, and others don't" won't have a direct impact on that conclusion.

Hope that is helpful. Please follow up if you have further questions, or if I haven't addressed your concerns, and good luck studying!
User avatar
 
WaltGrace1983
Thanks Received: 207
Atticus Finch
Atticus Finch
 
Posts: 837
Joined: March 30th, 2013
 
 
trophy
Most Thanked
trophy
Most Thankful
trophy
First Responder
 

Re: Q15 - Grow-Again ointment is

by WaltGrace1983 Thu Mar 20, 2014 3:18 pm

I thought that the answer to this would be much more complex, something like "one tube is all that is needed for one to get sufficient use out of the ointment." This would provide further reasoning why offering a rebate won't do much - the people who bought it don't need to buy anymore of it! However, the LSAT turned the tables on me and went a step further by talking about the people that haven't bought it yet.

Here is a further breakdown of this question:

Five drops daily is the recommended dose, more will not increase effectiveness
→
A rebate will not increase sales and would be unprofitable.

(C) If the rebate still wouldn't attract more purchases to the product, then it makes more sense that there isn't going to be a spike in sales and it also makes sense that it could be an unprofitable pursuit.

(D) We don't care about the causes of baldness. This has no bearing on the argument core whatsoever.

(E) This also seems to be a bit out of scope. We don't know anything about if these products will be produced in "large quantities."
 
ranishulman
Thanks Received: 0
Vinny Gambini
Vinny Gambini
 
Posts: 2
Joined: September 17th, 2015
 
 
 

Re: Q15 - Grow-Again ointment is

by ranishulman Tue Oct 27, 2015 2:23 pm

"They've given us information that hints at the fact that current customers will not buy more ointment (doesn't help to go beyond the recommended dose). Therefore, the company can probably only increase sales if they attract new customers." (Quoted from one of the posts).

Why are we assuming that current customers will not buy more ointment by the fact that applying more than the recommended dose will be ineffective?? Whose to say they won't buy more because it is "on-sale"??

With this logic, they may not be able to increase sales, but surely they won't be unprofitable. And who knows, since it is an effective product the current customers might supply their homes because they will save money in the long-run.
 
LoveLSAT
Thanks Received: 0
Vinny Gambini
Vinny Gambini
 
Posts: 1
Joined: July 26th, 2015
 
 
 

Re: Q15 - Grow-Again ointment is

by LoveLSAT Sun Nov 29, 2015 10:09 pm

For some reason, when I see this question, the first idea that entered in my mind is: five drops daily, umm, maybe somebody gets healed in one day, but others gets healed in one week. So, really, in order for the conclusion (rebate won't increase the sales) to be properly drawn, I will have to say something like everybody get the same amount of effectiveness on this ointment. then, I attempted to choose B), which actually weaken the conclusion.

Could you explain what is wrong with my reasoning above?
User avatar
 
maryadkins
Thanks Received: 641
Atticus Finch
Atticus Finch
 
Posts: 1261
Joined: March 23rd, 2011
 
 
 

Re: Q15 - Grow-Again ointment is

by maryadkins Mon Dec 07, 2015 11:33 am

So you would say that it would strengthen the argument to say, everyone is affected at the same rate by Grow-Again.

I guess I don't see how this would strengthen the conclusion about the rebate. Why would a rebate then be unprofitable? It could still draw more people to buy the product.

I think your reasoning here makes sense for poking holes in the premise, which is a stupid premise, but it doesn't touch on the conclusion.